How Do You Determine the Angle of Rotation from a 2x2 Matrix?

Click For Summary
To determine the angle of rotation from the given 2x2 matrix, the user analyzes the elements based on the standard rotation matrix format. The calculations yield angles of 126.9 degrees and 53.1 degrees from the inverse cosine and sine functions. However, there is confusion regarding the overall transformation, as adding these angles suggests a total of 180 degrees, which contradicts the expected matrix form. The user concludes that the angle of rotation from the positive x-axis is likely 126.9 degrees, but expresses uncertainty about the correct interpretation of the matrix. Clarification on the proper form of a rotation matrix is advised to resolve the discrepancies.
Natasha1
Messages
494
Reaction score
9
I just wanted to know how I could work out the angle of rotation from the following matrix -0.6, -0.8 (top) and 0.8, -0.6 (bottom)? :frown:

Is this possible or am I missing something here?
 
Physics news on Phys.org
Rotation matrices have a certain form (cos, sin, -sin, cos) or something. You can do inverse sin, or cos, so all is well.
 
right,

so I get:

a (top left of matrix) => cos-1 (-0.6) = 126.9 degrees

b (top right of matrix) => -sin-1 (-0.8) = 53.1 degrees

c (bottom left of matrix) => sin-1 (0.8) = 53.1 degrees

d (bottom right of matrix) => cos-1 (-0.6) = 126.9 degrees

What is the overall transformation? Is is 126.9 + 53.1 = 180 degrees?
 
It can't be 180 degrees, since that would give you a matrix of (-1, 0 , 0, -1), in the order, top left, top right, bottom left and bottom right, i think. I think that the angle of rotation from the positive x-axis is 126.9 degrees, but I'm not entirely sure.
 
It's either 123.9 or 56.1 (and of course they add up to 180), check the correct form of a rotation matrix in your notes.
 
Question: A clock's minute hand has length 4 and its hour hand has length 3. What is the distance between the tips at the moment when it is increasing most rapidly?(Putnam Exam Question) Answer: Making assumption that both the hands moves at constant angular velocities, the answer is ## \sqrt{7} .## But don't you think this assumption is somewhat doubtful and wrong?

Similar threads

Replies
2
Views
2K
Replies
10
Views
2K
  • · Replies 1 ·
Replies
1
Views
2K
Replies
2
Views
2K
  • · Replies 2 ·
Replies
2
Views
3K
  • · Replies 10 ·
Replies
10
Views
2K
  • · Replies 3 ·
Replies
3
Views
2K
  • · Replies 6 ·
Replies
6
Views
2K
  • · Replies 4 ·
Replies
4
Views
2K
  • · Replies 3 ·
Replies
3
Views
4K